Bayes theorem and probability help

Click For Summary
The discussion revolves around applying Bayes' theorem to a problem involving a car's oil pressure warning light. The light has a 99% probability of flashing when oil pressure is low and a 2% chance of flashing erroneously when oil pressure is normal. Participants clarify that the probabilities can exceed 100% because they pertain to different scenarios—one when oil pressure is low and another when it is not. The calculation for the probability of concern when the light flashes is suggested as P(concerned) = (0.99 x 0.1) + (0.02 x 0.9), resulting in a value of 0.117. This highlights the application of the law of total probability in solving the problem.
hoddo
Messages
5
Reaction score
0

Homework Statement


'A dashboard warning light is supposed to flash red if a car’s oil pressure is
too low. On a certain model, the probability of the light flashing when it should is 0.99; 2%
of the time, though, it flashes for no apparent reason. If there is a 10% chance that the oil
really is low, what is the probability that a driver needs to be concerned if the warning light
goes on?'


Homework Equations



Law of total probability/bayes theorem?


The Attempt at a Solution


having trouble making a start..
 
Physics news on Phys.org


Perhaps try drawing a tree diagram to help represent the situation.
 


i'm not sure how to do that though,
because of the 0.99 and the 2%,
add up to more than 1?
 


They are for different cases though, which is why it adds up to more than 100%.

The question states that IF the oil pressure is actually too low, then it will flash 99% of the time.

On the other hand though, if the oil pressure is not too low, then it will still flash 2% of the time.

Can you see how those figures are allowed to add up to more than 100%? Does that help?
 


P(concerned) = (0.99 x 0.1) + (0.02 x 0.9) = 0.117?
 
Question: A clock's minute hand has length 4 and its hour hand has length 3. What is the distance between the tips at the moment when it is increasing most rapidly?(Putnam Exam Question) Answer: Making assumption that both the hands moves at constant angular velocities, the answer is ## \sqrt{7} .## But don't you think this assumption is somewhat doubtful and wrong?

Similar threads

  • · Replies 2 ·
Replies
2
Views
1K
  • · Replies 1 ·
Replies
1
Views
2K
  • · Replies 5 ·
Replies
5
Views
4K
  • · Replies 2 ·
Replies
2
Views
6K
  • · Replies 4 ·
Replies
4
Views
5K
  • · Replies 4 ·
Replies
4
Views
3K
Replies
1
Views
1K
  • · Replies 1 ·
Replies
1
Views
2K
Replies
5
Views
2K
  • · Replies 1 ·
Replies
1
Views
3K